Tensor determinant using box product

Click For Summary
The discussion focuses on deriving the determinant of a tensor using index notation, specifically addressing two parts of a homework problem. Part (a) demonstrates that the Levi-Civita symbol multiplied by the determinant equals a specific product of matrix elements. In part (b), the challenge lies in manipulating the expressions to isolate the determinant, with participants noting the importance of recognizing the implications of specific indices in the Levi-Civita symbol. A key insight is that explicit indices like 1, 2, and 3 can only arise from specific assignments, not arbitrary ones. The conversation emphasizes the simplicity of the solution once the correct approach is recognized.
hotvette
Homework Helper
Messages
1,001
Reaction score
11

Homework Statement


Using index notation only (i.e. don't expand any sums) show that:
\begin{align*}
&\text{(a) } \epsilon_{ijk} \det \underline{\bf{A}} = \epsilon_{mnp} A_{mi} A_{nj} A_{pk} \\
& \text{(b) } \det \underline{\bf{A}} = \epsilon_{mnp} A_{m1} A_{n2} A_{p3}
\end{align*}

Homework Equations



\begin{align*}
&\text{(1) }\epsilon_{ijk} \det \underline{\bf{A}} = [\underline{\bf{A}} \underline{e}_i, \underline{\bf{A}} \underline{e}_j, \underline{\bf{A}} \underline{e}_k] \\
& \text{(2) } \underline{\bf{A}} = A_{ij} \underline{e}_i \otimes \underline{e}_j \\
& \text{(3) } \underline{\bf{A}} \underline{e}_j= (A_{pq} \underline{e}_p \otimes \underline{e}_q)\underline{e}_j
= (\underline{e}_q \cdot \underline{e}_j) A_{pq} \underline{e}_p = \delta_{qj} A_{pq} \underline{e}_p = A_{pj} \underline{e}_p
\end{align*}

The Attempt at a Solution


I got part (a) by using (3) for each term of the box product, substituting each of those expressions into the box product, and evaluating.

What baffles me is (b). If I multiply \epsilon_{ijk} times (b) and compare to (a), the net result is:
\begin{align*}
&\epsilon_{mnp} (\epsilon_{ijk} A_{m1} A_{n2} A_{p3}) = \epsilon_{mnp} (A_{mi} A_{nj} A_{pk}) \\
& \epsilon_{ijk} A_{m1} A_{n2} A_{p3} = A_{mi} A_{nj} A_{pk}
\end{align*}

But I have no idea what to do with that (the indices look really goofy). Any tips?
 
Last edited:
Physics news on Phys.org
hotvette said:
\begin{align*}
&\epsilon_{mnp} (\epsilon_{ijk} A_{m1} A_{n2} A_{p3}) = \epsilon_{mnp} (A_{mi} A_{nj} A_{pk}) \\
& \epsilon_{ijk} A_{m1} A_{n2} A_{p3} = A_{mi} A_{nj} A_{pk}
\end{align*}
You can't go from the first line to the second line by dividing by ##\epsilon_{mnp}## throughout - because there is an summation involved.

To arrive at (b) from (a), consider the value of ##\epsilon_{123}## :wink:
 
Ah, thanks. I was applying standard algebra techniques, which I now see is a mistake. Gosh, the answer is soooooo simple! Thanks.

P.S. I'd fiddled with this thing for probably 2 hours. At one point I was on the right track by looking at how to eliminate \epsilon_{ijk} from (a) to isolate the determinant. I thought about multiplying both sides by \epsilon_{ijk} to get 6 \det(A) = xxx but that didn't look promising. Drat!
 
Last edited:
Haha, I guess the trick is to realize that the only way you can have explicit labels such as 1,2 and 3 appearing in the indices is by setting them to be so - multiplying by other stuff with arbitrary indices can never give you specific indices.
 
Question: A clock's minute hand has length 4 and its hour hand has length 3. What is the distance between the tips at the moment when it is increasing most rapidly?(Putnam Exam Question) Answer: Making assumption that both the hands moves at constant angular velocities, the answer is ## \sqrt{7} .## But don't you think this assumption is somewhat doubtful and wrong?

Similar threads

  • · Replies 3 ·
Replies
3
Views
2K
  • · Replies 12 ·
Replies
12
Views
3K
  • · Replies 10 ·
Replies
10
Views
6K
  • · Replies 4 ·
Replies
4
Views
2K
  • · Replies 1 ·
Replies
1
Views
2K
  • · Replies 6 ·
Replies
6
Views
3K
  • · Replies 4 ·
Replies
4
Views
14K
  • · Replies 2 ·
Replies
2
Views
2K
  • · Replies 1 ·
Replies
1
Views
5K
  • · Replies 3 ·
Replies
3
Views
2K